Integral Ques: Show $\int_0^{\infty} x^4 \exp{-x^2} dx = \frac{3\sqrt{\pi}}{8}$

  • Thread starter Thread starter John O' Meara
  • Start date Start date
  • Tags Tags
    Integral
Click For Summary
The integral $\int_0^{\infty} x^4 e^{-x^2} dx$ is shown to equal $\frac{3\sqrt{\pi}}{8}$ using integration by parts and properties of Gaussian integrals. The discussion emphasizes the importance of correctly identifying the anti-derivative of the integrand, as the indefinite integral does not equal the definite integral provided. A substitution related to kinetic theory is also explored, linking the integral to the root mean square velocity of molecules. Participants suggest rewriting the integral and using appropriate substitutions to resolve issues with constant factors. The conversation highlights the intricacies of integration techniques and their applications in physics.
John O' Meara
Messages
325
Reaction score
0
Given that \int_{0}^{\infty} \exp{-x^2}dx =\frac{\sqrt{\pi}}{2} \\ \ \mbox{ show that } \ \int_0^{\infty} x^4 \exp{-x^2} dx= \frac{3\sqrt{\pi}}{8} \\. From the kinetic Theory the root mean square velocity of the molecules (\overline{v^2})^{\frac{1}{2}} is the square root of the integral 4\pi \frac{m}{2\pi kT} \int_0 ^{\infty} \exp{\frac{-mv^2}{2kT}}v^4 dv, where k is Boltzmann constant. T is the absolute temperature, m the mass of each molecule and v the speed of any molecule. Using the substitution x^2=\frac{mv^2}{2kT} \\ \ \mbox{ show that (\overline{v^2})^\frac{1}{2} = (\frac{3kT}{m})^\frac{1}{2} \\ Using integration by parts I get the following:<br /> \int x^4\exp{-x^2}dx =\lim_{x\rightarrow \infty} x^4|\frac{\sqrt{\pi}}{2} - \lim_{X\rightarrow \infty}\int 4x^3 dx \\ \ \mbox{ which does not give} \ 3\frac{\sqrt{\pi}}{8}. Any help would be welcome.<br /> <h2>Homework Statement </h2><br /> <h2>Homework Equations</h2><br /> <h2>The Attempt at a Solution</h2>
 
Last edited:
Physics news on Phys.org
The missing integral is: 4 \pi \frac{m}{2 \pi kT} \int_{0}^{\infty} \exp{\frac{-mv^2}{2kT}}v^4 dv
 
NO, NO, NO!

You are given that the definite integral:
\int_{0}^{\infty}e^{-x^{2}}=\frac{\sqrt{\pi}}{2}

When you do integration by parts, you need to find AN ANTI-DERIVATIVE of the integrand that you are to evaluate between the two limits.

The anti-derivative (a function!)\int{e}^{-x^{2}}dx is NOT equal to the value of the definite integral you were given!

Thus, you must proceed differently:
Let us rewrite:
\int_{0}^{\infty}x^{4}e^{-x^{2}}dx=\frac{1}{2}\int_{0}^{\infty}x^{3}(2xe^{-x^{2}})dx
Now, set:
u(x)=x^{3}, \frac{dv}{dx}=2xe^{-x^{2}}

See if you can use this hint properly..
 
Thanks for the reply.
\mbox{ Let } \ v=\int2x\exp{-x^2}dx \ \mbox{ therefore } \ v=-\int\exp{u}du = -\exp{u}du \ \mbox{ but } u=-x^2 \ \mbox{therefore } \ v=-\exp{-x^2} \\
For the second part x=v(\frac{m}{2kT})^{\frac{1}{2}} \ \mbox{ therefore } \ dv=dx(\frac{2kT}{m})^{\frac{1}{2}} \ \mbox{ and also therefore } \ v^3=x^3(\frac{2kT}{m})^{\frac{3}{2}} \\ \mbox{ and } v=\sqrt{\frac{2kT}{m}}.The result I get then is \frac{1}{2}(\frac{1}{2})^{\frac{3}{2}}\frac{3kT}{m}\\ which is out by the constant factor \frac{1}{2}(\frac{1}{2})^{\frac{3}{2}}\\.Thanks for the help.
 
Actually the missing integral is 4\pi(\frac{m}{2\pi kT})^{\frac{3}{2}}\int \mbox{ e.t.c.}
 
Question: A clock's minute hand has length 4 and its hour hand has length 3. What is the distance between the tips at the moment when it is increasing most rapidly?(Putnam Exam Question) Answer: Making assumption that both the hands moves at constant angular velocities, the answer is ## \sqrt{7} .## But don't you think this assumption is somewhat doubtful and wrong?

Similar threads

  • · Replies 105 ·
4
Replies
105
Views
6K
  • · Replies 5 ·
Replies
5
Views
2K
  • · Replies 47 ·
2
Replies
47
Views
4K
Replies
3
Views
2K
  • · Replies 34 ·
2
Replies
34
Views
3K
  • · Replies 5 ·
Replies
5
Views
3K
  • · Replies 7 ·
Replies
7
Views
3K
  • · Replies 3 ·
Replies
3
Views
2K
Replies
7
Views
2K
  • · Replies 3 ·
Replies
3
Views
2K